Legal Reasoning Flashcards

1
Q

Astha, who is a very good painter, is also a patient in a mental asylum, who, at intervals, is of sound mind. During one of these intervals, she entered into a relationship with Lopamudra to paint a picture of her for a specified amount. She, however, asked Lopamudra to pay her the entire amount in advance. One month later, on the day of delivery of the painting, Astha refused to perform the contract saying that she suffers from insanity. Can Lopamudra force performance?

Principle: A person is said to be of sound mind for the purpose of making a contract if, at the time when he makes it, he is capable of understanding it and of forming a rational judgment as to its effect upon his interests.

(a) Yes, because Astha was of sound mind when she entered into the contract.
(b) No, because Astha had been of unsound mind even while the contract was signed which is proved by the fact that she was admitted in an asylum.
(c) Yes, because a good painter can paint irrespective of his/her mental stability.
(d) No, because it was silly on Lopamudra’s part to enter into a contract with a mental patient admitted in an asylum.

A

(a) Yes, because Astha was of sound mind when she entered into the contract.

How well did you know this?
1
Not at all
2
3
4
5
Perfectly
2
Q

Principle: Mere silence as to the facts likely to affect the willingness of a person to enter into a contract is not a fraud, unless the circumstances of the case are such that, on close examination it is found to be the duty of the person keeping silent to speak, or unless his silence is, in itself, equivalent to speech.

Facts: X sells by auction to Y, a horse which X knows to be of unsound state of mind. X says nothing to Y about the horse’s unsound state of mind. Give the correct answer.

(a) X can be held liable for fraud.
(b) X can be held liable for misrepresentation.
(c) X cannot be held liable, because he did not say anything positive about the mental state of the horse.
(d) X cannot be held liable because it is the buyer who must be aware of the things.

A

(c) X cannot be held liable, because he did not say anything positive about the mental state of the horse.

How well did you know this?
1
Not at all
2
3
4
5
Perfectly
3
Q

Agni enters into a contract with Tanuj whereby Tanuj will supply Agni with 10 grams of cocaine for a specified amount. Is the contract void?

Principle: If the consideration or object of an agreement is forbidden by law, or is of such a nature that would defeat the provisions of any law, or is fraudulent, or is injurious to the person or property of another or, the Court regards it as immoral, or opposed to public policy, then the object or consideration shall be deemed unlawful. Every agreement of which the object or consideration is unlawful is void.

a. Yes, because the contract is for the sale of illegal drugs.
b. No, because Agni and Tanuj have entered into the contract out of their own free will, and being the citizens of a free country, they have the right to do so.
c. Yes, because drugs are harmful.
(d) None of these.

A

a. Yes, because the contract is for the sale of illegal drugs.

How well did you know this?
1
Not at all
2
3
4
5
Perfectly
4
Q

If the tax rate increases with the higher level of income, it is called –

(a) Progressive Tax (b) Proportional Tax (c) Lump sum Tax (d) Regressive Tax

A

(a) Progressive Tax

How well did you know this?
1
Not at all
2
3
4
5
Perfectly
5
Q

A fake doctor operated on a man for internal piles by cutting them out with an kitchen ordinary knife. The man died of haemorrhage.

(a) Doctor is guilty of murder
(b) Doctor is not guilty.
(c) Doctor is guilty of culpable homicide not amounting to murder
(d) None of these.

A

C

How well did you know this?
1
Not at all
2
3
4
5
Perfectly
6
Q

Dr Mortimer performed a kidney operation upon James for removal of kidney stones. James was already affected by HIV. Dr Mortimer had warned James of all the possible risks. James, out of his own volition, decided to undertake the risks and signed a bond certifying the same. James died of haemorrhage as a result of the operation.

(a) Doctor is guilty of murder.
(b) Doctor is not guilty.
(c) Doctor is guilty of culpable homicide not amounting to murder
(d) None of these.

A

B

How well did you know this?
1
Not at all
2
3
4
5
Perfectly
7
Q

Principle: Only Parliament or State Legislatures have the authority to enact laws on their own.
No law made by the State can take away a person’s fundamental right.
Facts: Parliament enacted a law, which according to a group of lawyers is violating the fundamental rights of traders. A group of lawyers files a writ petition challenging the Constitutional validity of the statute seeking relief to quash the statute and further direct Parliament to enact a new law.
(a) No writ would lie against Parliament, as the court has no authority to direct Parliament toenact or re-enact a law
(b) The court can quash existing law if it violates fundamental rights and can direct Parliament tomake a new law
(c) The court can quash the existing law if it violates fundamental rights but cannot direct
Parliament to make a new law
(d) None of these

A

C

How well did you know this?
1
Not at all
2
3
4
5
Perfectly
8
Q

Principle: When one person signifies to another his willingness to do or abstain from doing anything, with a view to obtaining the assent of that person to such an act or abstinence, he is said to have made a proposal.
Fact: “Ramanuj telegraphed to Shyam Sunder, writing: “Will you sell me your Rolls
Royce CAR? Telegram the lowest cash price.” Shyam Sunder also replied by telegram: “Lowest price for CAR is Rs. 20 lakh.” Ramanuj immediately sent his consent through telegram stating: “I agree to buy the CAR for Rs. 20 lakh asked by you.” Shyam Sunder refused to sell the car.
(a) He cannot refuse to sell the CAR because the contract has already been made.
(b) He can refuse to sell the CAR because it was only invitation to offer and not the real offer
(c) It was not a valid offer because willingness to enter into a contract was absent
(d) None of these

A

B

How well did you know this?
1
Not at all
2
3
4
5
Perfectly
9
Q

Principle: A person is said to be of sound mind for the purpose of making a contract if, at the time when he makes it, he is capable of understanding it and of forming a rational judgment as to its effect upon his interests.
Facts: Mr. X who is usually of sound state of mind, but occasionally of unsound state of mind, enters into a contract with Mr. Y when he was of unsound state of mind. Mr. Y having come to know about this fact afterwards, wants to file a suit against Mr. X.
(a) Mr. X cannot enter into contract because he is of unsound state of mind when he entered intocontract.
(b) Mr. X can enter into contract but the burden is on the other party to prove that he was ofunsound state of mind at the time of contract.
(c) Mr. X can enter into contract but the burden is on Mr. X to prove that he was of soundstate of mind at the time of contract.

A

C

How well did you know this?
1
Not at all
2
3
4
5
Perfectly
10
Q

Principle: Willful rash driving is an offense.
Facts: Mr. Tiwari was driving his car after drinking alcohol. Police books him for willful negligent driving. Is the act of the police lawful?

(a) No, because Mr. Tiwari was not driving rashly; he was drunk while driving.
(b) No, this is not a negligent act.
(c) Yes, because Mr. Tiwari was driving rashly.
(d) Yes, because the police has the power to arrest a person driving rashly.

A

A

How well did you know this?
1
Not at all
2
3
4
5
Perfectly
11
Q

Principle: Ignorance of Fact is excused but ignorance of law is no excuse.
Fact: X was a passenger from Zurich to Manila in a Swiss Plane. When the plane landed at the Airport of Bombay on 28 Nov. 1962 it was found on searching that X carried 34 kg of Gold Bars on his person and that he had not declared it in the ‘Manifest for Transit’. On 26th Nov. 1962 the Government of India had issued a notification modifying its earlier exemption, making it mandatory now that the gold must be declared in the “Manifest” of the aircraft.
(a) X cannot be prosecuted because he had actually no knowledge about the new notificationissued two days ago
(b) X cannot be prosecuted because ignorance of fact is excusable
(c) X can be prosecuted because ignorance of law is not excusable
(d) X’s liability would depend on the discretion of the court

A

C

How well did you know this?
1
Not at all
2
3
4
5
Perfectly
12
Q

Principle: Any direct physical interference with goods in somebody’s possession without lawful justification is called trespass of goods.
Facts: Z purchased a car from a person who had no title to it and sent it to a garage for repair. X believing wrongly that the car was his, removed it from the garage.
(a) X cannot be held responsible for trespass of goods as he was under a wrong belief.
(b) X can be held responsible for trespass of goods (c) X has not committed any wrong.
(d) None of the above.

A

B

How well did you know this?
1
Not at all
2
3
4
5
Perfectly
13
Q

Principle: Mere silence as to the facts likely to affect the willingness of a person to enter into a contract is not a fraud, unless the circumstances of the case are such that, on close examination it is found to be the duty of the person keeping silent to speak, or unless his silence is, in itself, equivalent to speech.
Facts: X sells by auction to Y, a horse which X knows to be of unsound state of mind. X says nothing to Y about the horse’s unsound state of mind. Give the correct answer-
(a) X can be held liable for fraud
(b) X can be held liable for misrepresentation
(c) X cannot be held liable, because he did not say anything positive about the mental stateof the horse
(d) X cannot be held liable because it is the buyer who must be aware of the things

A

C

How well did you know this?
1
Not at all
2
3
4
5
Perfectly
14
Q

On 31st March 1976, FCRA was enacted with an aim to regulate the utilization of foreign contributions/hospitality by individuals, associations to keep it consistent with the values of a sovereign, democratic republic. The FCRA was enacted in 1976 in order to maintain strict control over voluntary organisations and political associations that received foreign fundings. In 1984, an amendment was made to the act requiring all the Non-Governmental Organisations to register themselves with the Home Ministry. In 2010, the act was repealed and a new act with strict provisions was enacted. FCRA 2010 is a consolidating act passed by the Government of India in the year 2010. It seeks to regulate foreign contributions or donations and hospitality (air travel, hotel accommodation etc) to Indian organizations and individuals and to stop such contributions which might damage the national interest. It is an act passed for regulating and prohibiting the acceptance and utilization of foreign contribution or foreign hospitality by companies, associations or individuals for such activities that could prove to be detrimental to the national interest and for matters connected therewith or incidental thereto. Since the Act is internal security legislation, despite being a law related to financial legislation, it falls into the purview of the Home Ministry and not the Reserve Bank of India. The foreign inflow has almost doubled in the last decade, however, as per the government, the entities receiving the funds aren’t using it for the declared purpose. In FCRA 2020, only 20% of the foreign funds can be used for administrative purposes while the limit was 50% in FCRA 2010. This might hamper the workings of several small NGOs who depend on such funds. The new provisions aim to enhance transparency and accountability in the matter of foreign funds inflow and utilisation. The bill also makes the Aadhar number mandatory for recipients (passport or OCI card will be used as the identification document in case of foreigners).

  1. FCRA 2010 regulates:
    A. Foreign Hospitality B. Foreign Contribution
    C. Both A and B.
    D. Neither: as they are regulated by FEMA
2. FCRA applies to: 
A.	Whole of India 
B.	Indian Citizens Outside India 
C.	Branches of Indian Registered Co’s outside India 
D.	All of these
A

1c

2d

How well did you know this?
1
Not at all
2
3
4
5
Perfectly
15
Q

Mr. Freeman of Delhi was gifted for his personal use, a pen worth 30,000/- by the organizers of the event which he was an invitee for: Is it a Foreign Contribution?

A. Yes, amount exceeds 25000/
B. Yes, contribution is made by a foreign source
C. No, is in nature of gift for personal use
D. No, as gift by a professional body not classified as foreign source

A

C

How well did you know this?
1
Not at all
2
3
4
5
Perfectly
16
Q

Which of these are Foreign Source:

A. Club exclusively made up of Indian Citizens, formed and registered outside India
B. Club exclusively made up of Indian Citizens, formed and registered inside India
C. Both a and b
D. Neither a nor b

A

A

How well did you know this?
1
Not at all
2
3
4
5
Perfectly
17
Q

Arnab, an editor of Never-ever Magazine, a bi-monthly registered publication magazine, with national circulation, took money from Foreign Source: Is this permissible u/s 3 of FCRA?
A. Yes, as it’s a magazine and not a registered newspaper
B. No, as no distinction is made between a Magazine or any other publication u/s 3 of FCRA
C. Yes, as it is not a daily issue and prohibition applies to daily issues only
D. No, as it is a nationally circulated magazine

A

A

How well did you know this?
1
Not at all
2
3
4
5
Perfectly
18
Q

In the instant case, the appealing party was a restricted organization. He was mainly in business of manufacturing and selling of sugars. On October 10th, 1968, a piece of news showed up expressing that the respondent-State of Uttar Pradesh had chosen to give exception from deals charge for a time of three years under Section 4-A, Uttar Pradesh Sales Tax Act, 1948 to all new modern units in the State of Uttar Pradesh. On October 11th, 1968 the appealing party kept in touch with the Director of Industries expressing that taking into account the business charge occasion reported by the administration, the litigant wanted to set up a plant for the production of vanaspati and looked for affirmation of the exception. The Director of Industries affirmed the position. An affirmation to a similar impact was given by the Chief Secretary, Government of Uttar Pradesh. Taking into account these confirmations, the appealing party proceeded with the setting up of the processing plant. In May 1969, the State administration of Uttar Pradesh reconsidered on the topic of exclusion and mentioned the litigant to go to a gathering. At the gathering, the appealing party’s agent said that on the affirmation and assurance of the respondent (the legislature of Uttar Pradesh) the litigant had continued setting up the manufacturing plant. In this belief that the Government has exempted him from tax, he took a huge loan and started to work on it. However, after sometime, the Government gave a second thought to its tax exemption policy. It called for a meeting regarding this issue and called the petitioner to attend the meeting. Petitioner sent his representative to attend the meeting. The State Government of Uttar Pradesh, in any case, on January 20th, 1970, took a strategy choice that new vanaspati units which went into business creation by September 30th, 1970, would be given a fractional concession of deals charge. The litigant’s factor went into creation on July 2nd, 1970, yet the State Government indeed changed its arrangement, and on August 12th, 1970 suggested its choice to cancel the
concessions.

  1. Whether the plaintiff waived his right to have a cause of action by accepting partial exemption?
    A. Government would be liable if the petitioner did not accept the concessional rates of tax. But he did waive his right to have a cause of action by accepting the partial exemption.
    B. There can be no waiver of the rights unless the person who waived it has all information and knowledge about his right and waiver of the same. Waiver should be an intentional act with knowledge. There was no reason to believe that appellant was aware that his right would be waived due to partial exemption.
    C. No, plaintiff didn’t waive his right to have a cause of action by accepting partial exemption.
    D. Yes, plaintiff waived his right to have a cause of action by accepting partial exemption.
    E. No writ petition can be entertained by court in this case
  2. Whether the plaintiff can have a cause of action on the grounds of promissory estoppel?
    A. Yes, in such cases contractual obligation is mandatory to bring a cause of action on the basis of promissory estopple.
    B. There is no need for any contractual obligations before bringing a cause of action on the grounds of promissory estoppels. Action could be brought in those cases where no contractual obligation exists, but the same is intended for the future
    C. No, Contractual obligation is not mandatory to bring a cause of action on the basis of promissory estopple.
    D. This is depending on the discretion of the court
  3. Whether any such action against the Government acting in governmental, sovereign or administrative capacity can lie?
    A. No, Absolutely Not, where the element of sovereign function is present, one cannot sue the govt.
    B. This doctrine would be applied against the Government, where it is necessary to prevent fraud and manifest justice. Where the Government owes a duty to the public to act differently, promissory estoppel cannot be invoked to prevent the Government from doing so.

C. Yes, such action against the Government acting in governmental, sovereign or administrative capacity can lie if it is permissible by the apex court.
D. Sovereign immunity of government is absolute and it cannot be challenge in any circumstance.

  1. Whether in the present case, the doctrine would be applicable because the plaintiff did not suffer any detriment?
    A. To invoke the doctrine of promissory estoppels, it is not necessary for the promise to show that he suffered detriment as a result of acting in accordance with promise.
    B. To invoke the doctrine of promissory estoppels, it is necessary for the promise to show that he suffered detriment as a result of acting in accordance with promise.
    C. No, because it is always mandatory to suffer losses to invoke doctrine of promissory estopple.
    D. This will be decided by the facts and circumstances and positions of parties to the case and it vary from case to case along with the discretion of the court.
10. Promissory estoppel is sometimes spoken of as a substitute for: 
A.	Novation 
B.	Quasi Contract 
C.	Coercion 
D.	Contract
A
6b
7b
8b
9a
10b
How well did you know this?
1
Not at all
2
3
4
5
Perfectly
19
Q

Passage 1:
Ralia Ram was one of its partners of a firm which dealt in bullion and other goods at Amritsar. It was duly registered under the Indian Partnership Act. On the 20th of September, 1947 Ralia Ram arrived at Meerut by the Frontier Mail to sell gold, silver, and other goods in the Meerut market. Whilst he was passing through the Chaupla Bazaar with this object, he was taken into custody by three police constables. His belongings were then searched and he was taken to the Kotwali Police Station. He was detained in the police lock-up there and his belongings were seized from him and kept in police custody. Then he was released on bail, and sometime thereafter the silver seized from him was returned to him. Ralia Ram made repeated demands for the return of the gold but he could not recover the gold from the police officers, he filed the present suit against the respondent in which he claimed a decree that the gold seized from him should either be returned to him or in the alternative, its value should be ordered to be paid to him. The gold in question had been taken into custody by one Mohammad Amir, who was then the Head Constable, and it had been kept in the police Malkhana under his charge. Mohd. Amir, however, misappropriated the gold and fled away to Pakistan soon thereafter. He had also misappropriated some other cash and articles deposited in the Malkhana before he left India. The respondent further alleged that a case under section 409 of the Indian Penal Code as well as s. 29 of the Police Act had been registered against Mohd. Amir, but he had not been apprehended. Alternatively, it was pleaded by the respondent that this was not a case of negligence of the police officers and that even if negligence was held proved against the said police officers, the respondent State could not be said to be liable for the loss resulting from such negligence.
1. Choose best available option:
A. State is responsible for the damage caused to Ralia Ram and must pay damages to him.
B. Constable Md. Amir is responsible as he done the act out of course of employment.
C. State is not responsible and will not pay the damages.
D. This is negligence on the part of Police department and department was acting under the state, so state is vicariously liable for the same.
E. Both State and Police department is liable for the damage caused to Ralia Ram.

A

C

How well did you know this?
1
Not at all
2
3
4
5
Perfectly
20
Q

The conductor invited passengers to travel on the roof of the bus. Because the bus was overloaded with passengers. To overtake a cart, the driver over turned the bus abruptly to right side of the road. As the driver turned on the kutcha portion of the road, a passenger names Tahir Sheikh, was hit by a branch of a tree and fell down from the bus and got seriously injured and later died. In an action by the bus driver, the mother of the deceased claimed compensation.
1. Choose best available option:
A. Absolute case of Volenti non fit injuria, no damages will be awarded.
B. Inevitable accident, as it is general trend in India to travel on the roof of bus.
C. Damages will be awarded after deducting the amount of contributory negligence.
D. Full damages will be awarded as it was the conductor who invited the passengers to travel on the roof of bus.
E. Full damages will be awarded as it was the conductor who invited the passengers to travel on the roof of bus and the driver who was driving negligently.

A

C

How well did you know this?
1
Not at all
2
3
4
5
Perfectly
21
Q

The working and management of college was carried by a society registered under Jammu and Kashmir Registration of Societies Act,1898. The Society is empowered by clause 3 sub- clause II of the Memorandum of Association to make rules for the conduct of the affairs of the Society and to, amend, them from time to time with the approval of the Government of Jammu and Kashmir and the Central Government. The college through a notice invited application for the B.E Course in various branches of Engineering. Petitioner applied for admission in the college. There were two sets of tests conducted Written Test and Viva Voice Test. The petitioner secured good marks in the written test but he was awarded poor marks in the viva examination due to which he didn’t clarify the admission test. The viva test conducted of the petitioner was of 2 to 3 minutes in which questions related to parenting and residence were asked. No formal question pertaining to the subject was asked to the petitioner. The petitioner found that there were few students who secured vey less marks in the written test but got selected for the admission in the college merely because they obtained high marks in the viva voice test. The petitioner aggrieved by the selection procedure of candidates in the Regional Engineering College filed a writ petitioner under Article 32 of the Constitution claiming that he was denied Right to Equality and the selection procedure was arbitrary as the selection criteria was relied on viva voice examination and by holding viva voice test lasting of only 2 or 3 minutes and asking questions which had no significance in changing the intellect of the candidate.
1. Choose best available option:
A. Writ petition is maintainable against the college because college is an instrumentality of state and selection procedure is violative of Article 14.
B. Writ petition is not maintainable against the college because college is not an instrumentality of state and selection procedure is not violative of Article 14.
C. Writ petition is not maintainable against the college because college is not an instrumentality of state but selection procedure is violative of Article 14.
D. Writ petition maintainable under article 226 only but the selection procedure is violative of Article 14.
Writ petition can be filed for the arbitrary selection procedure only.

A

A

How well did you know this?
1
Not at all
2
3
4
5
Perfectly
22
Q

On 11th November, 1970 at around 10:00 pm deceased Ranjit and his brother Samir were sitting outside their house and suddenly 3 police vehicles having 15-20 officers stopped in front of their house and rushed towards them. Both of them started running towards their house and the police officers were chasing them, having revolvers in their hands. Bibhuti Chakraborty made a shot from point blank range on Ranjit and other officers were also constantly firing on both of them which injured Ranjit and were able to catch both of them. They were dragged and dumped into the car. Ranjit and Samir both died before any medical assistance could be provided to them as they were injured due to the tussle at their house. Thereafter, Benoy brother of deceased was taken into custody, but was released on the bail. So, aggrieved by the decision of the police officers he filed a complaint in the court and the trial was initiated against them in the Sessions Court of Calcutta. The learned Chief Metropolitan Magistrate initiated proceedings against respondents and convicted them for the offence under Sec 302 read with Sec 34 of Indian Penal Code (I.P.C) and sentenced them to life imprisonment. An appeal was filed by the respondents in the High Court of Calcutta against the conviction order as passed by the subordinate court. The Honorable Court acquitted the appellants on the ground that police was on patrol duty and orders were there from Deputy Commissioner for open fire and thus actions of the convicts were totally justified. The court set aside the orders of trial court and acquitted the appellants for the case being. Aggrieved by the decision, the Special Leave Petition is filed by the State of West Bengal against the impugned judgment of the High Court of Calcutta dated 1st August, 1980.
1. Choose best available option:
A. SC will uphold the decision of trial court and convict them under Sec 302 and 34.
B. SC will uphold the decision of High Court and acquit all the accused
C. As the order to open the fire was given by superior authorities of police officers, so they cannot be charged for murder and hence SC will charge them for GH only.
D. As per the facts of case it is clearly established that deceased didn’t open fire on police party, it was police party who open the fire first without any apprehension of fear and death so the SC will uphold the decision of trial court and convict them under Sec. 302 and 34 of IPC, because this is not justified act.

A

B

How well did you know this?
1
Not at all
2
3
4
5
Perfectly
23
Q

A Hyena, wild animal was moving in the villages and causing injuries and deaths to small children. The people frightened with it. They complained to the authorities. The Government deputed certain officers. While they were wandering in the forests in search of Hyena, they saw a moving animal behind the bushes. It was a rainy day and the vision was not clear. The officers thought that it was the Hyena. It was common that no people would be moving in that area and in particularly in that rainy time. The accused, one of the officers, fired at the moving object. The result was that the death of a human being.
5. Choose best available option:
A. He will be convicted for death by Rash and Negligent Act.
B. He will be Convicted for Culpable homicide not amount to murder.
C. He is having absolute immunity of mistake of fact and working under good faith and will not be convicted.
D. It was just an accident and he will not be charged for the same and Court will acquit him accordingly.
E. Although he was working under the order of the authorities but the element of good faith was missing as other officers didn’t fire on that moving object. So, he will be liable for the CH not amount to murder

A

D

How well did you know this?
1
Not at all
2
3
4
5
Perfectly
24
Q
The defendants (the Board) agreed with certain landowners adjoining a stream to improve the banks of the stream and to maintain them in good condition. The landlords on their part paid proportionate costs. Subsequently one of the landlords sold his land to the first plaintiff and he to the second plaintiff. There was negligence on the part of the Board in maintaining the banks, which burst and the land was flooded. Both the plaintiffs were strangers to the agreement with the Board. 
5.	Choose Correct option in the light of doctrine of privity of contract: 
A.	Plaintiffs can sue the board. 
B.	Plaintiffs cannot sue the Board. 
C.	Only original landlord (who sold the land to plaintiff) can sue the board on the behalf of plaintiffs (beneficiary rule) 
D.	Both the Plaintiffs and Original landlord (all three) can sue the board 
E.	According to doctrine of privity of contract no one is in the capacity to sue the board, as original owner lost his right after selling the land to plaintiffs and Board didn’t contract with the plaintiffs.
A

A

How well did you know this?
1
Not at all
2
3
4
5
Perfectly
25
Q

State Bank of Patiala, subsidiary of State Bank of India, issued a circular No. PER/VRS/48 dated 20th January 2001 publishing their Voluntary Retirement Scheme (hereinafter referred to as ‘the SBPVRS’) drawn up in the light of the guidelines issued by the Indian Bank Association. The object of the SBPVRS inter alia was to downsize the existing strength of the employees and to increase profitability. The scheme was to open on February 15, 2001 and it was to close on 1st March, 2001 (inclusive of both days). The applications under the SBPVRS were to be accepted during the period when the scheme was to remain open between 15th February 2001 to 1st March 2001. The applications for Voluntary Retirement under the SBPVRS will be accepted during this period only. No voluntary retirement shall be deemed to have come into effect unless the decision of the Competent Authority has been communicated in writing, which will be convened within a maximum period of two months after the date of closure of receipt of applications i.e., 01.03.2001. The application once made cannot be withdrawn and the same will be treated as irrevocable. While making application, the employee will be required to declare the name of nominee, to whom the payment may be made in the event of death of an VRS opted after the competent authority has accepted his VRS application but before payment has been affected. To complete the chronology of events, respondents herein applied under the SBPVRS between 15th February 2001 and 1st March 2001. Respondent No.1 withdrew his application for voluntary retirement on 3/5th March, 2001. Similarly, respondent No. 2 withdrew his application for voluntary retirement on 2nd March 2001. Respondent No. 3 withdrew his application for voluntary retirement on 5th March, 2001. However, on 3rd April, 2001, the bank refused permission to the said respondents to withdraw from the scheme since their withdrawal was made after the date of the closure of the scheme on 1st March, 2001.
1. Choose best available option:
A. Employee can withdraw their application
B. Employee cannot withdraw their application
C. Depends on the discretion of the court
D. State Bank of India having statutory authority, so employee cannot withdraw their application.
E. It is well versed by the facts of the case that it was already informed by the bank that the once the application for VRS made cannot revoked after prescribed date, it was a clear communication of facts between the parties and their implied conduct showed that they agreed to this condition of the bank. So, there is no ground to challenge the decision of bank and court will reject the claim of employees.

A

A

How well did you know this?
1
Not at all
2
3
4
5
Perfectly
26
Q

An official of the appellant/defendant company, furnished information in the presence of some others those certain dealers were going to be employed by the I.B.P., one such being a dealer at a particular spot and encouraged the plaintiff/respondent that he being a local, a local man had better prospects for the job and satisfied the criteria laid down by the company for its illegibility. Relying on the information that particular dealer done all the required arrangement in pursuant of that particular offer and his implied conduct showed that he accepted the offer. But later on company didn’t employed that dealer. He filed suit against the company.
2. Choose best available option:
A. Company is liable for breach of contract and has to pay damages to dealer
B. Company is not liable for breach of contract and will not pay damages
C. Official of the company is liable
D. Information furnished by official is amount to invitation to offer, hence no contract, so no breach of contract and company is not liable.
E. There is not contract but court will award damages to that dealer as he made some arrangements in pursuant of the offer he accepted.

A

B

How well did you know this?
1
Not at all
2
3
4
5
Perfectly
27
Q

Mr and Mrs Merritt married in 1941. They held their matrimonial home in joint names. In 1966 Mr Merritt left the family home to live with another woman as he grows the love with that woman. Mr Merritt agreed to pay Mrs Merritt £40 per month. At Mrs Merritt’s request, he signed a document confirming that when she had repaid the balance on the mortgage, he would transfer the matrimonial home into her sole name. Mrs Merritt paid off the mortgage and successfully acquired a declaration that the house belonged to her. Mr Merritt appealed. Mr Merritt contended the agreement was a domestic arrangement between husband and wife and there was no intention to create legal relations and, as such, there was no enforceable contract. He also argued the purported contract was insufficiently certain to be enforceable by the court, and that Mrs Merritt had failed to provide consideration for his promise.
3. Choose best available option:
A. Mr. Merritt appeal will be successful as there was no intention to create legal relationship between the parties.
B. Mr. Merritt appeal will be successful as there was not sufficient consideration on the part of Mrs Merritt.
C. Mr. Merritt appeal will be successful as it was a domestic agreement between husband and wife with other flaws like intention to create legal relationship and insufficient consideration, that amounts to no agreement in eye of law.
D. Mr. Merritt appeal will be not be successful as he left his wife and grow love with other women (dill diya galla)
E. Mr. Merritt appeal will be not be successful as Ms. Merritt’s claim possess all essential requirements.

A

E

How well did you know this?
1
Not at all
2
3
4
5
Perfectly
28
Q

Consider the following statements regarding Fundamental Duties:
A. A person should respect the fundamental rights and duties equally because in any case, if the court comes to know that a person who wants his/her rights to be enforced is careless about his/her duties then the court will not be lenient in his/her case.
B. Any ambiguous statute can be interpreted with the help of fundamental duties.
C. The court can consider the law reasonable if it gives effect to any of the fundamental duties. In this way, the court can save such law from being declared as unconstitutional.
D. One can be punished on violation of certain fundamental duties. Being non justiciable they are justiciable otherwise E. Fundamental Duties can override Fundamental Rights.

4.	Choose best available option: 
A.	All Statements are wrong 
B.	All statements are correct 
C.	Only A is Correct 
D.	Only A and B is Correct 
E.	Only A B and D are Correct
A

B

How well did you know this?
1
Not at all
2
3
4
5
Perfectly
29
Q

Consider the following statements regarding as principle of sovereign immunity of the State for the acts of its servants.
A. The State is not liable for torts committed by its employees within the scope of their employment, if the tort is committed in connection with the exercise of sovereign functions of the State and the mere fact that a tort is committed by a member of the defense forces does not make the activity a sovereign one.
B. The State is liable for torts committed by its employees within the scope of their employment, if the tort was committed in connection with the exercise of the non-sovereign functions of the State.
C. Welfare activities of the government are not regarded as ‘sovereign functions’ for this purpose nor are routine activities, such as maintenance of vehicles of officers of the government.
D. A ‘service’ (facility) provided to a ‘consumer’ within the meaning of the Consumer Protection Act, 1986 is not a ‘sovereign’ function. Running of a hospital is also not a sovereign function.
5. Choose Correct option:
A. Only A B and C are Correct
B. Only A and B are Correct
C. Only A B and D are Correct
D. Only A Correct
E. All are correct

A

E

How well did you know this?
1
Not at all
2
3
4
5
Perfectly
30
Q

Since Criminal Intention is the basis of criminal liability, an inebriated person, is in the same state of mental condition as an insane person. Such state of mind has been termed dementia affectatia – a form of lunacy in which the function of the mind is temporarily suspended.
Consider the following statement in the light of above statement:
A. Above statement refers to Involuntary intoxication.
B. The justification for such a provision is based on the contention that the accused had not contributed himself towards his drunkenness and which he is not likely to be repeated as in the case of voluntary act.
C. It is said that one who sins when drunk, should be punished when he is sober.
D. But if a man is forced to drunkenness through trick, fraud or ignorance, without the accused’s knowledge or against his will, the act is not voluntary act and so he is excused from liability.
Choose the Correct option:
A. A and C are Correct
B. B, C and D are Correct
C. D, A and B are Correct
D. All are correct

A

C

How well did you know this?
1
Not at all
2
3
4
5
Perfectly
31
Q

There was a quarrel between the accused and the deceased and the accused whipped out a knife and stabbed the accused on the chest near the shoulder. The stab injury was not on a vital part of the chest, but since the knife cut the artery inside, it resulted in death. Accused would be liable for…….
Choose best available option:
A. Murder
B. Culpable homicide not amounts to murder
C. Death by Rash and Negligent Act
D. Grievous Hurt

A

B

How well did you know this?
1
Not at all
2
3
4
5
Perfectly
32
Q

The plaintiff, an Abkari Contractor, who was in default on the payment of Toddy Welfare and was ineligible to participate in the auction, and to take any contract in his name. He agreed with the defendant that the defendant should take some shops at the auction and transfer some of these shops to the plaintiff. The plaintiff paid some consideration to the defendant for the same. The defendant having failed to perform his part of the agreement, the plaintiff claimed back the consideration paid by him to the defendant.
Choose best available option:
A. Contract is Valid and made by free consent of parties, hence defendant has to paid back the consideration to plaintiff.
B. Contract is Void because no sufficient consideration as he has to participate himself in auction.
C. Contract is void ab initio and defendant is not liable for breach of contract
D. Contract is Void ab initio but as per law the defendant is liable for breach of contract
E. As per basic principles of law, there is valid contract between plaintiff and defendant because when plaintiff was ineligible to participate in auction and he didn’t participate in the same and allow defendant to participate in auction and later on they had valid and mutual contract to transfer the shops according to their free consent. So, in this whole transaction there are two valid contracts. First one is that defendant applied for the auction and he got the same, and second one is when defendant has to transfer his ownership to plaintiff. So, both contracts are valid and free from any technical issue.

A

C

How well did you know this?
1
Not at all
2
3
4
5
Perfectly
33
Q

The defendant owed a sum of money under a promissory note to his father. The defendant perpetually, day and night, complained to his father that he had not been treated equally with other children in the distribution of his property. Thereupon the father promised to discharge him from all liability in respect of the loan and the note, provided he would stop complaining, which the defendant accordingly did. The question was whether the defendant’s promise to cease his complaints was a sufficient consideration to sustain his father’s promise?
Choose best available answer:
A. Yes
B. May be, depends on party
C. Depends on the party relationships
D. It would be ridiculous to suppose that such promises could be binding. In reality there was no consideration whatever.” “A contract founded upon such an illusory consideration appears to be as invalid as a promise by a father made in consideration that his son would not bore him”
E. This case falls under the doctrine of ultimate judicial absolute immunity, according to which such exceptional cases on consideration can only be decided by the discretion of the court and equity.

A

D

How well did you know this?
1
Not at all
2
3
4
5
Perfectly
34
Q

The Plaintiff and defendant, who married under Mohammedan law, agreed before marriage that the defendant (wife) would be allowed to live with her parents after the marriage. The wife went to her parents and refused to come back to her husband (Plaintiff). He filed the suit for restitution of conjugal rights. Her defence was that she was permitted by the agreement made before marriage, to live apart and that husband had not paid dower amounting to rupees 1000. It is to be noted here that In Islamic law marriage is a civil contract between parties which allows them mutually to agree upon the terms and conditions of their future together. Like any other contract, the free consent of the parties to the agreement to marry is essential.
Choose best available option:
A. Court will allow her to live with her parents.
B. Court will ask her to come back to her husband.
C. Court will allow them to end the marriage if they don’t want to live together.
D. Court will not validate the agreement which they had before marriage and order for restitution of conjugal rights.
E. It is to be noted here that as marriage is civil contract in Islamic law and parties had the contract with free consent, so court will validate the agreement and ask them if they don’t want to continue the marriage then they can go for separation through legal process(divorce).

A

D

How well did you know this?
1
Not at all
2
3
4
5
Perfectly
35
Q

A, was a specialized and famous ship manufacture and he was famous for his best designs and manufacturing quality of ships and on various occasions he also received awards and recognitions for the same. One day Mr. B contacted him and gave an order to build a ship for fishing in the sea. B gave specification for the ship and Mr. A build the ship as per given specifications. When Mr. B put it in use and the ship turned unfit for fishing in the sea. Mr. B sued Mr. A for the damages.
Choose best available option:

A. It’s a goodwill of Mr. A, which motivated Mr. B to contract with him for ship manufacturing and Mr. A was unable to deliver the required ship because Mr. B already give some specifications and direction to Mr. A for ship manufacturing and Mr. A failed. Hence, he is liable to pay damages to Mr. B

B. It is true that Mr. B was attracted to award winning performances and recognition of Mr. A in ship manufacturing but it was his duty to keep regular or routine checks on his customized ship building process, so that any fault can be avoid during the manufacturing process itself and Mr. B failed to do that. Hence, he is not liable for damages.

C. As Mr. A was an experienced ship manufacturer so it was his duty to tell Mr. B regarding the defects in specifications given by Mr. B but Mr. A concealed them. It was breach of duty to take care, hence Mr. Ais liable to pay damages to Mr. B

D. Mere silence as to facts likely to affect the willingness of a person to enter into a contract is not fraud, unless the circumstances of the case are such that, regard being had to them, it is the duty
of the person keeping silence to speak, or unless his silence, is, in itself, equivalent to speech. Hence Mr. Ais liable to pay damages to Mr. B.

E. This contract is void ab intio between Mr. A and Mr. B as there is no consensus ad idem between the parties. Because the ship demanded and ship received was two different things.

A

B

How well did you know this?
1
Not at all
2
3
4
5
Perfectly
36
Q

In this case Principal authorized his agent by power of attorney to sell one of his properties only (item number 1). The powers of attorney were tampered and interpolated to include another property (Item No. 2) by agent and subsequently sold off by the agent to third party. Later on, Principal sued third party for the recovery of second property (Item No. 2).
Choose best available option:

A. Tempering the power of attorney was illegal on behalf of agent and out of the course of employment as he was not authorized to sell second property. Hence principal will recover his second property.

B. Agent was authorized to sold off the item no. 1 and not item number 2 and he played the fraud with both his principal and third party so agent is liable for the fraud and principal will not be able to recover the item number 2 from third party.

C. In this case the first contract of item number 1 is valid and executable as it has good consideration and agent was having the proper authority from the principal but the second contract of item number 2 is beyond the powers of agent and out of the course of employment of agent and he also acted fraudulently,
so court will validate the first contract and second contract will be void ab initio and principal will recover his property.

D. The whole transaction is void ab intio as agent acted out of his course of employment and tempered and interpolated the power of attorney. So, court will declare the whole transaction null and void.

A

B

How well did you know this?
1
Not at all
2
3
4
5
Perfectly
37
Q

The plaintiff engaged the defendant estate agents to sell a property, instructing them to market it at 6,500 pounds but that he would accept 6,000 pounds. The plaintiff accepted an offer of 6,150 pounds ‘subject to contract’. Before exchange, another potential buyer offered 6,750 pounds. Instead of communicating that offer to their principal, the agents went to the original offeror, suggesting he could sell on and make a profit. They did so in good faith, believing that they had already fulfilled their duty to their principal. Later on Principal came to know about this fact and sued the agent for the loss suffered by him:
Choose best available option:

A. Principal received more than what he asked for. So no damages.

B. In this case Principal had a contract with an agent to sold his property on prescribed amount and the agent did the same and the act of the agent to sold the property later via original offeror to make profit is all together a different contract. So, agent cannot be mad liable. Moreover, it is his job to sell and purchase the property being an agent. So, no damages.

C. Principal will get the damages as agent didn’t fulfill his duty properly and conceal the facts from principal.

D. By the implied and express conduct of principle it is clearly visible that he was ready to sell the property in 6000 pound and he instructed the agent to market the property for 6500 pounds so he can get at least 6000 pounds by negotiations. But agent in good faith and by best of his skill arranged a buyer who offered 6150 pounds for the said property. So, agent full fill his duty accordingly and he is not liable to pay the damages to principal.
So, the principal suit will be rejected.

A

C

How well did you know this?
1
Not at all
2
3
4
5
Perfectly
38
Q

The Plaintiff was an employee in the London Passenger Transport Board. The Board gave a free pass to plaintiff to travel free of charge on its omnibus subject to a condition that the Board or its servant would not be liable for any injury caused during the travel. Due to the Negligence of the defendant, a bus driver of the Board, the Plaintiff suffered injuries. Since he could not hold the Board liable in view of the exemption clause the plaintiff brought an action against the defendant personally recover the damages.
Choose best available option:

A. Plaintiff will not get any damages as it was already mentioned in the exemption clause the board or his servant will not be liable for any injury.

B. Plaintiff will not get damages as after deducting the contributory negligence as after agreeing to exemption clause he choose to travel in the omnibus.

C. Volenti non fit injuria will applicable here. No damages.

D. There is no enough consideration to create a liability as he was travelling free of cost.

E. Defendant is liable to compensate the plaintiff because being not a party to the contract the exemption clause did not cover his liability.

A

E

How well did you know this?
1
Not at all
2
3
4
5
Perfectly
39
Q

Both the Plaintiff and defendant went to horse race in Madras. They put an equal sum of the money (total 400 Rs.) on the same horse in the name of the defendant. The horse won the race. The defendant collected the winnings but did not pay the half share of the plaintiff.
Plaintiff sued the defendant.
Choose best available option:

A. Plaintiff will not get half amount of winnings.

B. Plaintiff will not get half amount of winnings.

C. Wagering agreement, expressly declared void by sec 30.

D. This suit is not maintainable in the court of law.

A

C

How well did you know this?
1
Not at all
2
3
4
5
Perfectly
40
Q

The Sultan of Johore residing in England as a private person, made a
promise of marriage to young women. Later he returned to native
state breaking his promise. She filed a suit for enforcing the promise.
Choose best available option:
A. When one visit to foreign country, he or she by their implied
conduct submit themselves to the jurisdiction of the law of that
country. So, in this case the lady can sue the Sultan for marriage
B. She cannot sue the Sultan for the enforcement of his promise.
C. Sultan ha to marry her or pay the compensation to lady for
breach of contract.
D. Sultan was residing in capacity of private person not as sultan
and he also promised her in his private capacity so he is bound
by promise.
E. As Sultan is a foreign national the court can not compel him to
fulfill his promise but the court will direct him to pay the
damages to lady for the breach of contract.

A

b

How well did you know this?
1
Not at all
2
3
4
5
Perfectly
41
Q

An unruly mob was heading toward Police and destroying the public
property. It was a very aggressive mob and destroying every
property, vehicles, shops whatever comes in its way. The mob
including person of every age from child to old age persons. People
who were standing on the roofs of their houses were also hooting
and shouting like anything. Police fired in the air to disperse the
mob. Plaintiffs who were standing on the roof of their houses
sustained injuries from bullets fired in the air. They sued the Police
for damages.
Choose best available option:
A. Sovereign immunity to Police as it is sovereign function to
maintain the law and order in the state.
B. Police officer who fired the bullet will be liable
C. State will be liable and pay the damages.
D. No damages as being a prudent man the persons standing on
their roof know the fact that police are firing to control the mob
so they can leave the palace.
E. Contributory negligence principle will apply here and damages
will be awarded accordingly.

A

c

How well did you know this?
1
Not at all
2
3
4
5
Perfectly
42
Q

. Who has been recently appointed the new Chairman of the Insurance Regulatory and Development Authority of India (Irdai)?

a. Mukesh Narain
b. Mahesh Jain
c. Deepak Gupta
d. Debasish Pand

A

d

How well did you know this?
1
Not at all
2
3
4
5
Perfectly
43
Q

Which Indian metro city’s first Climate Action Plan (MCAP) was released recently?

a. New Delhi
b. Kolkata
c. Mumbai
d. Chennai

A

c

44
Q

Which Indian cricketer recently broke the legendary Kapil Dev’s 40-year-old record of fastest Test fifty by an Indian?

a. Mayank Aggarwal
b. Rohit Sharma
c. Rishabh Pant
d. Ravindra Jadeja

A

c

45
Q

Recently, The Union Cabinet was apprised of a Memorandum of Understanding signed between the Indian Council of Medical Research and the____________.

a. Oxford University
b. Cambridge University
c. Stanford University
d. Cardiff University

A

a

46
Q

Every year _________________is celebrated as the World Consumer Rights Day.

a. 10th March
b. 12th March
c. 15th March
d. 18th March

A

c

47
Q

On September 2, the defendants sent a letter offering to sell quantity of wool to the plaintiffs. The letter added “receiving your answer in course of post”. The letter reached the plaintiffs on September 5. On that evening the plaintiffs wrote an answer agreeing to accept the wool. This was received by the defendants on September 9. The defendants waited for the acceptance up to September 8 and not having received it, sold the wool to other parties on that date. They were sued for breach of contract. It was contended on their behalf that till the plaintiffs’ answer was actually received there could be no binding contract and, therefore, they were free to sell the wool on 8th.
Choose best available option:
A. Defendant is not liable for breach of Contract
B. Defendant is liable for breach of Contract
C. Defendant is liable for Anticipatory breach of Contract
D. Invitation to offer, not binding and hence defendant is not liable
E. The defendants were not bound by their offer when accepted by the plaintiffs till the answer was received, then the plaintiffs ought not to be bound till after they had received the notification that the defendants had received their answer and assented to it.

A

B

48
Q

The accused was a compounder in a small hospital. The senior doctor of the hospital arrived in the morning with a bad headache and asked the accused for 10 grams of aspirin. The accused took 12-13 minutes to bring the aspirin, which was readily available in the dispensing room. The doctor consumed the medicine. It was bitter, which was an unusual taste for aspirin. He asked the attender to fetch him a glass of water. By then, the second doctor was sitting in the next chair. The senior doctor complained about the strange bitterness in the tongue, though aspirin was supposed to be tasteless. He gargled, washed his face with water and asked the attendant to buy some betel leaves to overcome the bad taste. He thereafter proceeded to do his normal work and tried to give injection to a waiting patient but began to feel shaky. He had sensation of cramps in the calf muscles. The other doctor ran into the dispensing room and asked the accused from which bottle he had given the aspirin. The accused showed him the aspirin bottle. The doctor asked him if he had given strychnine, a deadly poison accidentally. The accused denied it, stating that strychnine was not in stock at all. He started trembling. In the meantime, the doctor was rushed to the hospital and given a stomach wash. It was found that it was indeed strychnine, which was administered to the doctor by the accused.
Choose best available option:
A. Compounder liable for Culpable homicide amount to murder
B. Compounder liable for Culpable homicide not amount to murder
C. Compounder liable for Voluntarily causing grievous hurt
D. Compounder is liable for causing hurt with intention
E. Compounder is liable for causing simple hurt

A

D

49
Q
An assistant superintendent of commercial taxes paid a surprise visit to the shop of the accused to inspect the books of account. He found two sets of account books in the shop. He took them and started looking into them. Suddenly, the accused snatched away both the books from him. Decide the liability of accused. 
Choose best available option: 
A.	Assault  
B.	Criminal Force 
C.	Battery 
D.	None of the above  
E.	Assault or Criminal Force
A

E

50
Q

The defense of necessity finds place in Penal laws of almost all the countries influenced by the common law. However, the defense does not give blanket immunity from liability in all circumstances. It is only available only in exceptional cases.
Consider the following statement in the light of above statement.
A. Where the act was done in order to avoid consequences which could not otherwise be avoided, and which, if they had followed, would have caused irreparable damage.
B. That no more harm was done than was necessary for that purpose.
C. That the evil inflicted was not disproportionate to the evil avoided.
D. The harm was done to avert greater evil and other means were less effective, and the means employed were more efficient.

A

D

51
Q

15th November 1940, Bonda Kui and her niece were the sole residents of the house. In the middle of the night, she saw a figure dancing in a state of complete nudity with a broomstick tied on one side and a torn mat around the waist. The woman thought it is an evil spirit which consumes humans and so she gave repeated blows by a hatchet and felled the thing to the ground. She immediately informed her niece that she had killed an evil spirit. Examination showed, however, that she had killed a human being who was the wife of her husband’s brother. The only evidence is this case were the statements made by the appellant from time to time. All along she maintained that she did not take the unfortunate deceased to be a human being at all. She thought that it was something which eats up human beings.
Court Held:
The Sessions Judge convicted her under Section 304 of IPC. However, keeping in the mind the state of society to which she belonged and her superstitious beliefs, the Judge imposed upon her a sentence of six years rigorous imprisonment only. Thus, the Additional Sessions
Judge convicted fifty years old Bonda Kui only for an offence under Section 304 of IPC and sentenced her to imprisonment. Choose Correct option: The Conviction of Bonda Kui was………………
A. Justified
B. Not Justified
C. Not appropriate as sentence awarded was less D. She don’t have any immunity in IPC

A

B

52
Q

Article 245: Extent of laws made by Parliament and by the
Legislatures of States.
(1) Subject to the provisions of this Constitution, Parliament may make laws for the whole or any part of the territory of India, and the Legislature of a State may make laws for the whole or any part of the State
(2) No law made by Parliament shall be deemed to be invalid on the ground that it would have extra territorial operation. This implies that state law cannot have extra-territorial operation.
A live in Rajasthan running a Lottery business in Bangalore using his network and people over there. The people in Bangalore will pay for lottery tickets and from whatever receipts A get, A will pay the prize money and keep rest as profit after paying applicable taxes. Since A is living in Rajasthan but his business was conducted in Bangalore, should A pay tax to Karnataka Government?
Choose best available option:
A. A will pay tax to Rajasthan only as he is resident of Rajasthan only.
B. Since A distributed all the prize money in Bangalore only so he is not liable to pay tax to Karnataka govt.
C. A should pay tax to Karnataka Government.
D. A should pay tax to both Rajasthan and Karnataka.

A

C

53
Q

Under Union List, Entry 2A, deployment of armed forces come under the purview of Central Government. In-State List, Entry 1, makes the ‘public order’ aspect under the State List. Can State make a law for public order, which involves deployment of armed forces?
Consider the following Statements:
A. State can enact a law under public order in which state can deploy armed forces in case of riots and armed rebellion.
B. State cannot enact a law related to deployment of armed forces as it is given in union list.
C. State can enact a law related to deployment of armed forces by the permission of Central Government.
D. Only A and C
E. None of the Above

A

B

54
Q

In the case of State of Rajasthan v. ………………., the state of Rajasthan made a law restricting the use of sound amplifiers. This law was violated by the respondent and the act was impugned by the judicial magistrate. On further appeal to the Supreme Court, the state argued that the law was within the legislative competence of the state legislature under the entry 6 of List II, i.e. the power to legislate in relation to public health includes the power to legislate in relation to public health includes the power to regulate the use of amplifier as it produces loud noise whereas the opposition argued that amplifiers came under entry 31 list I i.e. post and telegraphs; telephones; wireless; broadcasting and other like forms of communication.
Choose best available option:
A. The law made by state of Rajasthan is Constitutionally valid.
B. The law made by state of Rajasthan is Unconstitutional as it is matter of Union List.
C. The law will become Constitutional if the center govt. approved it. As it is matter of Union list.
D. Doctrine of Colorable Legislation, invalid law
E. None of the Above

A

A

55
Q

Consider the following statements regarding Trespass:
A. Trespass to a person in itself consist of three torts.
B. As a genus it comprises trespass to the person, trespass to moveable’s and trespass to land.
C. Trespass can be termed as the wrongful act done directly and intentionally to cause the harm to another property can be termed as trespass.
D. It is of two kinds of civil and criminal wrong.
E. It is an act intentionally directed for an unreasonable interference with the one’s property and persons.The term intentionally here implies that any wrongful act done voluntarily.

Choose Correct option: 
A.	Only B C D and E 
B.	Only B and E 
C.	Only B D and E 
D.	Only A B C D and E 
E.	Only E and B
A

D

56
Q

Consider the following statements regarding Judicial Review:
A. Our constitution contains express provisions for judicial review of legislation as to its conformity with the constitution.
B. As long as some Fundamental Rights exist and are a part of the Constitution, the power of judicial review has also to be exercised with a view to see that the guarantees afforded by these Rights are not contravened
C. The court held that judicial review is a ‘basic feature’ of the constitution and it could not be taken away by putting a law under the Ninth Schedule.
D. All amendments to the Constitution made on or after 24th April, 1973 by which the Ninth Schedule is amended by inclusion of various laws therein shall have to be tested on the touchstone of the basic or essential features of the Constitution as reflected in Article 21 read with Articles 14 and 19 and the principles underlying them.
E. Judicial review can be of administrative action of the Union and State and authorities under the state.

Choose correct option: 
A.	None of the Above 
B.	Only A B C D and E 
C.	Only A B C and D 
D.	Only B C and D 
E.	Only A B C and E
A

B

57
Q

The plaintiff, an Abkari Contractor, who was in default on the payment of Toddy Welfare and was ineligible to participate in the auction, and to take any contract in his name. He agreed with the defendant that the defendant should take some shops at the auction and transfer some of these shops to the plaintiff. The plaintiff paid some consideration to the defendant for the same. The defendant having failed to perform his part of the agreement, the plaintiff claimed back the consideration paid by him to the defendant.
Choose best available option:
A. Contract is Valid and made by free consent of parties, hence defendant has to paid back the consideration to plaintiff.
B. Contract is Void because no sufficient consideration as he has to participate himself in auction.
C. Contract is void ab initio and defendant is not liable for breach of contract
D. Contract is Void ab initio but as per law the defendant is liable for breach of contract
E. As per basic principles of law, there is valid contract between plaintiff and defendant because when plaintiff was ineligible to participate in auction and he didn’t participate in the same and allow defendant to participate in auction and later on they had valid and mutual contract to transfer the shops according to their free consent. So, in this whole transaction there are two valid contracts. First one is that defendant applied for the auction and he got the same, and second one is when defendant has to transfer his ownership to plaintiff. So both contracts are valid and free from any technical issue.

A

C

58
Q

The defendant owed a sum of money under a promissory note to his father. The defendant perpetually, day and night, complained to his father that he had not been treated equally with other children in the distribution of his property. Thereupon the father promised to discharge him from all liability in respect of the loan and the note, provided he would stop complaining, which the defendant accordingly did. The question was whether the defendant’s promise to cease his complaints was a sufficient consideration to sustain his father’s promise?
Choose best available answer:
A. Yes
B. May be, depends on party
C. Depends on the party relationships
D. It would be ridiculous to suppose that such promises could be binding. In reality there was no consideration whatever.” “A contract founded upon such an illusory consideration appears to be as invalid as a promise by a father made in consideration that his son would not bore him“
E. This case falls under the doctrine of ultimate judicial absolute immunity, according to which such exceptional cases on consideration can only be decided by the discretion of the court and equity.

A

D

59
Q

Principle: A defamatory statement is one which has a tendency to injure the reputation of the person to whom it refers to. Defamation is of two kinds:
Libel, when the defamatory statement is in some permanent and visible form.
Slander, when it is in some transitory form, visible or audible. Libel is actionable per se, but slander is actionable only on proof of actual damage.
Factual Situation: A person accused of a crime is arrested by the police. The police give an open statement that the suspect had an extra-marital affair. It is subsequently proved in court that the suspect was innocent of the crime. Will the accused succeed if he files a civil suit for defamation, claiming compensation against the police?
A. He will succeed as the statement is defamatory.
B. He will not succeed as the police officer is doing his duty in finding a motive for the crime.
C. He will not succeed as it was a bonafide allegation made by the police in the course of the investigation
D. He will succeed if he can prove actual damage caused to his professional and/or personal life.

A

D

60
Q

The Plaintiff and defendant, who married under Mohammedan law, agreed before marriage that the defendant (wife) would be allowed to live with her parents after the marriage. The wife went to her parents and refused to come back to her husband (Plaintiff). He filed the suit for restitution of conjugal rights. Her defence was that she was permitted by the agreement made before marriage, to live apart and that husband had not paid dower amounting to rupees 1000. It is to be noted here that In Islamic law marriage is a civil contract between parties which allows them mutually to agree upon the terms and conditions of their future together. Like any other contract, the free consent of the parties to the agreement to marry is essential.
Choose best available option:
A. Court will allow her to live with her parents.
B. Court will ask her to come back to her husband.
C. Court will allow them to end the marriage if they don’t want to live together.
D. Court will not validate the agreement which they had before marriage and order for restitution of conjugal rights.
E. It is to be noted here that as marriage is civil contract in Islamic law and parties had the contract with free consent, so court will validate the agreement and ask them if they don’t want to continue the marriage then they can go for separation through legal process(divorce).

A

D

61
Q

Principle: By virtue of s. 330 of the Code, if a person voluntarily causes hurt for the purpose of extorting confession from the sufferer or any information which may lead to the detection of an offence, he shall be punishable with imprisonment of either description for a term which may extend to seven years and shall also be liable to fine.
Factual Situation: X, a police officer tortures Y, to tell him where the stolen property was kept by him. Has A committed any offence?
A. X is not liable since he is discharging his duty.
B. X is liable since the custodial torture is not part of duty C. X is not liable since the torture was to extract confession.
D. None of the Above

A

B

62
Q

A lady granted/ gifted a property consisting of some land to her daughter (defendant) by a gift deed. The deed was registered to the proper authorities. One of the terms of the deed was that the daughter had to pay a sum of Rs.653 annually. Later the old lady died, and the defendant refused to pay the money the sister whom she had promised to pay so. And hence the plaintiff sued the defendant for the recovery of the same. Issue: Whether the plaintiff can bring an action against the defendant for the amount promised in a contract where the consideration for such promise has been furnished by the mother of the defendant (plaintiff’s sister)?
Appellants Argument: The consideration for getting the property was a promise to pay the amount annually to the plaintiff.
Respondent’s Argument: The Plaintiff was not party to contract, and hence no right to compel the compel respondent for paying the promised amount.
Choose best available option:
A. Consideration has to be paid by respondent to Appellant
B. Respondent has no need to pay the consideration to appellant
C. Respondent no need to pay the consideration as per doctrine of privity of contract
D. The gift deed and the annuity agreement are both simultaneous agreements and both can be considered as one transaction and thus, consideration has to be paid by respondent to Appellant.

A

D

63
Q

Consider the following statements regarding the term Justice as used in Preamble:
A. The term ‘justice’ in the Preamble embraces three distinct forms— social, economic and political, secured through only various provisions of Directive Principles.
B. Social justice denotes the equal treatment of all citizens without any social distinction based on caste, color, race, religion, sex and so on. It means absence of privileges being extended to any particular section of the society, and improvement in the conditions of backward classes (SCs, STs and OBCs) and women.
C. Economic justice denotes the non-discrimination between people on the basis of economic factors. It involves the elimination of glaring inequalities in wealth, income and property.
D. A combination of social justice and economic justice denotes what is known as ‘distributive justice’. Political justice implies that all citizens should have equal political rights, equal access to all political offices and equal voice in the government.
E. The ideal of justice—social, economic and political—has been taken from the French Revolution (1917).
Choose Correct Option:
A. Only A B C and E are correct
B. Only A B and C are Correct
C. Only A and B are correct.
D. All are correct
E. Only B C and D are correct

A

D

64
Q

A democratic polity, as stipulated in the Preamble, is based on the doctrine of popular sovereignty, that is, possession of supreme power by the people.
Consider the following statements in the light of above statement:
A. In indirect democracy, the people exercise their supreme power indirectly as is the case in Switzerland. There are four devices of indirect democracy, namely, Referendum, Initiative, Recall and Plebiscite.
B. In direct democracy, on the other hand, the representatives elected by the people exercise the supreme power and thus carry on the government and make the laws. This type of democracy, also known as representative democracy, is of two kinds—parliamentary and presidential.
C. The Indian Constitution provides for representative parliamentary democracy under which the executive is responsible to the legislature for all its policies and actions. Universal adult franchise, periodic elections, rule of law, independence of judiciary, and absence of discrimination on certain grounds are the manifestations of the democratic character of the Indian polity.
D. The term ‘democratic’ is used in the Preamble in the broader sense embracing not only political democracy but also social and economic democracy.
Choose correct option:
A. Only C and D are correct
B. Only A B and C are Correct
C. Only A and B are correct.
D. All are correct

A

A

65
Q

Consider the following statements regard Secular Character of Indian Polity:
A. Secularism means “State shall not have any official religion”.
B. The term ‘secular’ too was added by the 42nd Constitutional Amendment Act of 1976. However, as the Supreme Court said in 1974, although the words ‘secular state’ were not expressly mentioned in the Constitution.
C. There can be no doubt that Constitution-makers wanted to establish such a state and accordingly Articles 25 to 28 (guaranteeing the fundamental right to freedom of religion) have been included in the constitution.
D. The Indian Constitution embodies the negative concept of secularism i.e., all religions in our country (irrespective of their strength) have the same status and support from the state.
Choose correct option:
A. Only A and C are correct
B. Only A B and C are Correct
C. Only A and B are correct.
D. All are correct

A

B

66
Q

Consider the following statements regarding Indian Socialism i.e. the word Socialist use in preamble.
A. Even before the term was added by the 42nd Amendment in 1976, the Constitution had a socialist content in the form of certain Directive Principles of State Policy. In other words, what was hitherto implicit in the Constitution has now been made explicit.
B. Indian brand of socialism is a ‘democratic socialism’ and not a ‘communistic socialism’ (also known as ‘state socialism’) which involves the nationalization of all means of production and distribution and the abolition of private property.
C. Democratic socialism, on the other hand, holds faith in a ‘mixed economy’ where both public and private sectors coexist side by side. ‘Democratic socialism aims to end poverty, ignorance, disease and inequality of opportunity. Indian socialism is a blend of Marxism and Gandhism, leaning heavily towards Gandhian socialism’.
D. The new economic policy (1991) of liberalization, privatization and globalization has, however, diluted the socialist credentials of the Indian State.
Choose correct option:
A. Only A and C are correct
B. Only A B and C are Correct
C. Only A and B are correct.
D. All are correct
E. None of the Above

A

D

67
Q

Consider the following statements regarding emergency:
A. If approved by both the Houses of Parliament, the President’s Rule continues for six months 6.It can be extended for a maximum period of three Years with the approval of the Parliament, every six months.
B. However, if the dissolution of the Lok Sabha takes place during the period of six months without approving the further continuation of the President’s Rule, then the proclamation survives until 30 days from the first sitting of the Lok Sabha after its reconstitution, provided the Rajya Sabha has in the meantime approved its continuance.
C. Every resolution approving the proclamation of President’s Rule or its continuation can be passed by either House of Parliament only by a simple majority, that is, a majority of the members of that House present and voting.
D. A proclamation of President’s Rule may be revoked by the President at any time by a subsequent proclamation. Such a
proclamation does not require the parliamentary approval.

Choose Correct option: 
A.	Only A,D are Correct 
B.	Only C,A and B are Correct 
C Only A Correct 
D. All are Correct
A

D

68
Q

Consider the following statements regarding emergency:
A. While a proclamation of national emergency is in operation, the President can issue ordinances on the subjects mentioned in union list only.
B. Article 358 deals with the suspension of other Fundamental Rights
(except those guaranteed by Articles 20 and 21). while Article 359 deals with the suspension of the Fundamental Rights guaranteed by Article 19.
C. While a proclamation of national emergency is in operation, the state is freed from the restrictions imposed by Article 19.
D. Article 359 extends to the entire country whereas Article 358 may extend to the entire country or a part of it.

Choose Correct option: 
A.	Only A,D and C are Correct 
B.	Only C is Correct 
C.	None of the Above 
D.	All are Correct
A

B

69
Q

Principle: A defamatory statement is one which has a tendency to injure the reputation of the person to whom it refers to. Defamation is of two kinds-

(A) Libel, when the defamatory statement is in some permanent and visible form and
(B) Slander, when it is in some transitory form, visible or audible. Libel is actionable per se, but slander is actionable only on proof of actual damage.

Factual Situation: A person accused of a crime is arrested by the police. The police give an open statement that the suspect had an extra-marital affair. It is subsequently proved in court that the suspect was innocent of the crime. Will the accused succeed if he files a civil suit for defamation, claiming compensation against the police?

(a) He will succeed as the statement is defamatory
(b) He will not succeed as the police officer is doing his duty in finding a motive for the crime.
(c) He will not succeed as it was a bonafide allegation made by the police in the course of the investigation
(d) He will succeed if he can prove actual damage caused to his professional and/or personal life.

A

D

70
Q

bus driver was driving a bus along the BRT corridor in New Delhi. At a Bus stand he carelessly drove the bus resulting in hitting a railing and crushing a Marshall on duty at the spot. A lady standing at a distance, on hearing about the accident rushed to the spot and saw the injured covered in blood, hanging on a piece of the railing. The sight terrified the lady and as a result she fainted and had to be treated for nervous shock. The lady on recovery filed a suit for compensation against the driver and the owner of the bus.

(a) She will succeed as the accident sight was really shocking and anybody would have collapsed on seeing it.
(b) She will succeed only against the driver and not against the owner of the bus who did not foresee the accident.
(c) She will not succeed as the driver could not have foreseen the illness of the lady who came to the site.
(d) She will not succeed as she voluntarily went to the accident site

A

C

71
Q

Consider the following statements regarding Article 250.

A. A law made by Parliament which Parliament would not but for the issue of a Proclamation of Emergency have been competent to make shal, to the extent of the incompetency, cease to have effect on the expiration of a period of six months after the Proclamation has ceased to operate.
B. Council of States has declared by resolution supported by not less than two thirds of the members present and voting that it is necessary or expedient in national interest that Parliament should make laws with respect to any matter enumerated in the State List.
C. A resolution passed shall remain in force for such period not exceeding one year as may be specified therein.
D. it shall be lawful for Parliament to make laws for the whole or any part of the territory of India with respect to that matter while the resolution remains in force
Choose correct option
A. Only A Correct
B. Only B Correct
C. Only C, B and D Correct
D. All are correct

A

A

72
Q

An invasive therapeutic or investigative procedure without consent is technically a battery (trespass) which can be tried either under criminal or tort (civil) law. When it is a criminal offence, the indictments are framed under the penal code and when compensation is involved, tort law is used.

A. The Indian Penal Code (IPC) is not specific about the age at which a person can give valid consent. At present, the minimum legal age to give valid consent to undergo medical treatment is ambiguous.
B. Section 87 of the IPC says that a person above 18 years of age can give valid consent and Section 89 of the IPC says that a child under 12 years of age cannot give a valid consent.
C. However, the Indian Contract Act (Sec 11) states that every person who is of the age of majority is competent to contract.
D. According to the Indian Majority Act (Sec 3 (1)) every person attains the age of majority on his completing the age of 18 years.

Choose Correct option: 
A. All are correct 
B.A, B and Care Correct 
C.B, C and D are correct 
D.A, C and D are correct
A

A

73
Q

It is well settled practice and principle that before seizure of the property, concern taxation authority will send the notice (as required/s 154) to the concerned person defaulting tax. The appellant gave a blow to an inspector of Delhi Municipal Corporation who, in discharge of his duty, had gone to collect the milk tax (as provided under section 161 of the Corporation Act) and had seized the buffalo. The appellant was convicted for assaulting the inspector. Consider the following statements:
A. Appellant was acting under right to private defense of property.
B. Inspector was not acting in good faith as the due notice was not served before the seizure of property.
C. Inspector was acting in good faith and under the color of his office, though the act, may not be strictly justifiable by law.
D. Conviction of the appellant was not on merit and DMC is liable to pay the compensation to accused for its arbitrary action.

A

C

74
Q

A sub inspector of police accompanied by a constable, on being falsely informed that certain stolen property was in possession of one X, proceeded to his house. On arriving on X’s house he found that X was not at home so he demanded the said property from X’s wife, who denied any knowledge about the property in question and ask him to wait until the arrival of her husband. SI declined and threatened the women with the cane and laids hand on her. On hearing her cries, accused rushed to the spot to render help. An altercation ensued, and the accused on being assaulted by the sub inspector and constable snatched the heavy stick from the latter and struck two blows on the forehead of SI, which proved fatal.

Consider the following Statements:
A. Accused is guilty of murder of Sub Inspector as he was just investigating in good faith under the color of his office for the recovery of stolen property.
B. Accused is guilty of CH not amount to murder as SI was assaulted him and his wife and assault cannot be amount to taking the life of someone.
C. Accused is not guilty of any offence as he was just exercising his RPD.
D. Accused will be held only for causing Grievous hurt Voluntarily.

A

C

75
Q

Section 80 of IPC states, “nothing is an offense which is done by accident or misfortune and without any criminal intention or knowledge in the doing of a lawful act in a lawful manner by lawful means and with proper care and caution”. The act done should be by accident or misfortune.
The accused husband (A) and wife (B) and the deceased (C) were living in an apartment building. The couple had two Pit bull dogs aged 2 years. On the day of the fatal incident, A was out of town and left his wife in charge of dogs. B had taken one of her dogs to the roof of apartment building. While she was coming to her sixth floor of apartment, the dog attacked C, who had been returning to her apartment after shopping from grocery store. The 33 year old C suffered deep bites wound all over her body including neck and died after 5 hours later.

Défense taken by the A and B:

  1. B owned no responsibility to C’s death and said it to be an accident, she said she made a heroic effort to save C (it was evidentry as she also got some bite and scratches mark on her hand)
  2. A said he was out of town during the accident and that he had no way of foreseeing that either of their dogs was capable of deadly act.

Choose the correct option:
A. Both A and B will be liable under Section 304 and supporting sections of IPC that is Culpable homicide not amounting to murder.
B. Only B will be liable for the Culpable homicide not amounting to murder.
C. B will be liable for Murder of C
D. B will get the immunity of Section 80 i.e., “nothing is an offense which is done by accident or misfortune and without any criminal intention or knowledge.

A

A

76
Q

The defendant G, her half-sister C, and their mother A, all had history of heroin addiction, G brought three packages of heroin from a dealer T and handed over some or all of it to C, C injected herself while G and A were in the house, and subsequently developed and complained of symptoms consistent with an overdose. ( as it usually happened to her after taking an overdose). G and A decided not to take medical assistance and put c to be in recover position, hoping that she would recover spontaneously, G an A decided to sleep in the same room as C, C expected G and A to look after her need during the night. The following morning A woke and told G that C was dead. Cause of Death was over dose of heroine which was administered by C herself.
Choose the correct option:
A. G and A would not be liable for the death of C as whole family was addicted to heroin.
B. A would be liable for Murder as he supplied the heroine to C.
C. G and A would be held liable for manslaughter by gross negligence.
D. A would be held liable for Culpable homicide not amounting to murder.

A

C

77
Q

Section 89 - Nothing which is done in good faith for the benefit of a person under twelve years of age, or of unsound mind, by or by consent, either express or implied, of the guardian or other person having lawful charge of that person, is an offence by reason of any harm which it may cause, or be intended by the doer to cause or be known by the doer to be likely to cause to that person.
Consider the following statement in the light of above statement:
A. Immunity granted under this section is not absolute.
B. A child above 12 years of age is considered competent to give a consent for the purpose of IPC whereas a child below 18 years of age is not competent to give a valid consent for contract.
C. The harm caused under section 89 shall be for the benefit of a
person who is under 12 years of age.
D. This immunity shall not extend to the doing of anything which the person doing it knows to be likely to cause death, for any purpose other than the preventing of death or grievous hurt, or the curing of any grievous disease or infirmi-ty;

Choose the Correct option: 
A.	A and C are Correct 
B.	B, C and D are Correct 
C.	D, A and B are Correct 
D.	All are correct
A

D

78
Q

Consider the following statements regarding Fundamental Rights:
A. They constitute a very comprehensive economic, social and political programme for a modern democratic State. They aim at realizing the high ideals of justice, liberty, equality and fraternity as outlined in the Preamble to the Constitution.
B. They impose a moral obligation on the state authorities for their application, but the real force behind them is political, that is, public opinion.
C. As observed by Alladi Krishna Swamy Ayyar, ‘no ministry responsible to the people can afford light-heartedly to ignore the provisions in Part IV of the Constitution
D. Similarly, Dr B R Ambedkar said in the Constituent Assembly that ‘a government which rests on popular vote can hardly ignore the FR while shaping its policy. If any government ignores them, it will certainly have to answer for that before the electorate at the election time.
E. They embody the concept of a ‘welfare state’ and not that of a ‘police state’, which existed during the colonial era. In brief, they seek to establish economic and social democracy in the country.
Choose Correct option:
A. Only A B and D
B. Only B C and D
C. Only B and D
D. Only D
E. None of the above

A

E

79
Q

Consider the following statements regarding DPSP
A. Given in Part IV from Article 36-51.
B. They have served as useful beacon-lights to the courts. They have helped the courts in exercising their power of judicial review, that is, the power to determine the constitutional validity of a law.
C. They form the dominating background to all State action, legislative or executive and also a guide to the courts in some respects.
D. They facilitate stability and continuity in domestic and foreign policies in political, economic and social spheres in spite of the changes of the party in power.
E. They are supplementary to the fundamental rights of the citizens. They are intended to fill in the vacuum in Part III by providing for social and economic rights. Choose Correct option:
A. Only A
B. Only B
C. All of the above
D. Only A is correct rest options are regarding FR
E. None of the above

A

C

80
Q
Which of the followings are the DPSP? 
A.	Socialistic, Gandhian DPSP, Liberal Intellectual DPSP 
B.	Claims of SCs and STs to Services 
C.	Development of the Hindi Language 
D.	Instruction in mother tongue 
E.	Duty of the State to raise the level of nutrition and the standard of living and to improve public health 
Choose Correct option: 
A.	Only A  
B.	Only A and E 
C.	Only A D and E 
D.	Only A B C D and E 
E.	None of the above
A

D

81
Q

Consider the following statements:
A. Three-tier Panchayati raj system fall under ambit of DPSP.
B. Three-tier Panchayati raj system have element of
Constitutional Right.
C. Three-tier Panchayati raj system have element of Statutory Right.
D. Three-tier Panchayati raj system have element of Fundamental Rights.
E. Three-tier Panchayati raj system have element of Federal Structure.
Choose correct option:
A. Only D and B is Correct
B. Only D B and C are Correct
C. Only B is Correct
D. Only A B and E correct
E. All are correct

A

E

82
Q

Consider the following statements regarding Fundamental Duties:
A. They serve as a reminder to the citizens that while enjoying their rights, they should also be conscious of duties they owe to their country, their society and to their fellow citizens.
B. They serve as a warning against the anti-national and antisocial activities like burning the national flag, destroying public property and so on.
C. They serve as a source of inspiration for the citizens and promote a sense of discipline and commitment among them. They create a feeling that the citizens are no mere spectators but active participants in the realization of national goals.
D. They help the courts in examining and determining the constitutional validity of a law. In 1992, the Supreme Court ruled that in determining the constitutionality of any law
E. They are enforceable by law. Hence, the Parliament can provide for the imposition of appropriate penalty or punishment for failure to fulfill any of them. The importance of fundamental duties is that they define the moral obligations of all citizens to help in the promotion of the spirit of patriotism and to uphold the unity of India. Choose correct options:
A. Only A B and Care correct
B. Only A and C are Correct
C. Only A C D and E are correct
D. Only A B C D and E are correct
E. Only A B D and E are correct

A

D

83
Q

Principle: A plaintiff who suffers some injury will be entitled to receive compensation even if he suffers no loss.
Factual Situation: Reddy was a strong political worker of Party X. He was proceeding to the poling booth at about 7:30 am, to cast his vote in favor of
Mr. Naik, his party candidate. On the way, the State Police officials suspected
.Reddy to be an anti-social element and took him into custody for questioning. Reddy pleaded with the officials that he was a genuine voter and had no previous criminal records and requested them to allow him to cast his vote. The officials allegedly detained him in custody till about 4:45 pm and thereafter released him, recording that they could not find any thing against him. By the time Reddy reached the poling booth the polling time was over and he could not vote for Mr. Naik. When the results were declared Mr. Naik got a Majority of over 6000 votes and Party X got the majority to form the Government. Reddy files a suit for compensation against the State.
(a) Reddy will succeed as the police could not prove anything against him.
(b) Reddy will succeed as his right to vote was denied by the police
(c) Reddy will not succeed as his candidate won the election and therefore there is no cause of action against the police.
(d) Reddy will not succeed as the police can arrest a suspected criminal.

A

B

84
Q

Principle: The mere fact that a man is injured by another’s act gives in itselt no cause of action; if the act is deliberate, the party injured will have no claim in law even though the injury is intentional, so long as the other party is exercisinga legal right.”
Factual situation: Shanti Nikentan was a school situated in Jamnanagar of New delhi run by the plaintiff from last 10 years(from 2012). The defendant in 2022 set up a new school in the same locality because of which admissions in Shanti Niketan reduced. The plaintiff even had to reduce their fees and increase the salary of their teachers. Because of this the plaintiff suffered a huge financial loss. The plaintiff then filed a suit against the defendents for causing loss to the plaintiff .
Decide-
a.The defendents are liable because they caused damage to the plaintiff.
b.The defendents are liable because they have commited tort by causing damage to the plaintiff. It was their duty to not cause any loss to anyone in process of gaining profit for themselves.
c. The defendents are not liable because they didn’t knew about the existence of Shanti Nikentan. Had they knew about the school they would have been liable.
d.The defendents are not liable because even they’ve caused damage to the plaintiff , no legal right of the plaintiff was violated .

A

D

85
Q

Principle: Defamation is injury to the reputation of a person.If a person injures the reputation of another , he does so at his own risk , as in case of an interference with the property. A man’s property reputation is his property and if possible , more valuable, than other property.
Factual Situation:In D.P. Choudhary V. Manjulata, the plaintitf-respondent, Manjulata,about 17 years of age, belonged to a distinguished educated family of Jodhpur.She was a student of B.A. There was publication of a news item in a local daily, Dainik Navjyoti, dated 18.12.77 that last night at 11 p.m. Manjulata had run away with a boy named Kamiesh, after she went out of her house on the pretext of attending night classes in her college.
The news item was untrue and was published negligently with utter irresponsibility. She was shocked and ridiculed by persons who knew her marriage prospects were adversely affected.She thereby filed a case of defamation against D.P.Choudhary. Decide whether her claim for damages will succeed or not.
Choose the correct option-
a.The defendents are liable for slander as the words are defamatory and has caused harm to the reputation of the plaintiff.Hence plaintiffs are entitled to compensation.
b.The defendents are not liable because they were under mistake of fact and mistake of fact is good defence to the crime charged.
c.The defendents are liable because the defamatory words are actionable per se. It has caused harm to the reputation of the plaintiff and hence is entitled to compensation.
d.The defendents are not liable because the published statements did not refer to the plaintiff.The statement referred to the some other Manjulata.Hence no compensation to the plaintiff.

A

C

86
Q

Consider the following statements in light of amendment procedure in Indian Constitution-
1.Article 368 in Part XX of the Constitution deals with the powers of the parliament to amend the constitution and its procedure.
2.An amendment procedure can be initiated by the introduction of a bill for the purpose in either house of the parliament or in the state legislatures.
3.The bill can be introduced only by a minister that too with the prior permission of the president and not by any private member .
4.The bill must be passed in each house by a special majority , that is , a majority of the total membership of the members of the house present and voting
Choose the correct option-
a. Only 1 is correct
b. Only 1, 3 and 4 are correct
c. Only 1 and 4 are correct
4. All are correct.

A

C

87
Q

Mrs Verma married Mr Verma on 11 September 1880. After a year he deserted her on 13 December 1881. On inquiries made by her father and others she was led to believe husband had been drowned in a vessel bound for America, which went down with all hands (persons) on Board. On January 1887, Mrs Verma, supposing herself to be a Widow contracted a second marriage With another man. The circumstances were well known to the second husband and the ceremony was in no way concealed. The intent to commit bigamy was held to be negatived by a majority of five to nine judges by the accused’s mistaken belief in the death of her husband.
Choose the correct option-
a.Mrs. Verma is not liable because she was under mistake of fact. She in good faith believed that her husband was dead. She had sufficient grounds to believe in the death of her husband .
b.Mrs. Verma is liable for the offence of bigamy because even her spouse was dead she was not entitled to second marriage. As per hindu law, a person can marry only once in his lifetime.
c.Mrs. Verma is liable for bigamy because she did not make inquiry of her husband’s drowning by her own . She violated her duty to take care by believing on her father . She must have checked on her own if her father was saying the truth or not.
d.Mrs. Verma is not liable because she is a feminist women and she can marry whoever and whenever she wants because she has the fundamental right to life which includes right to marry as per one’s choice.

A

A

88
Q

Consider the following statements regarding President’s Rule on states:
A. It said that the power of the President to dismiss a government of a state is absolute.
B. It said that the President should use this power only after his proclamation (of imposing President’s Rule) has been approved by both Houses of the Parliament.
C. Until then, the President can only suspend the Legislative Assembly.
D. In case the proclamation does not get the approval of both the Houses, it lapses at the end of a period of two months, and the dismissed government is revived.
E. The suspended Legislative Assembly also gets reactivated. The SC also stated that the proclamation of the imposition of Article 356 is subject to judicial review. Choose correct option:
A. Only A B and Care correct
B. Only A and C are Correct
C. Only B C D and E are correct
D. Only A B C D and E are correct
E. None of the above

A

C

89
Q

The plaintiff was employed by a railway company to drill holes in a rock. Only nearby a crane was being operated by men employed by the railway company as well. The crane lifted stones and at times swung them over the plaintiff’s head without warning. The plaintiff was aware of the danger that he was exposed to by working near the crane. One time, a stone fell off the crane and injured the plaintiff.
He sued his employers for negligence.
Choose best available option:
A. Employer would be liable
B. Employer would not be liable
C. Clear cut case of Volenti non fit injuria
D. Employer would be liable + Contributory Negligence on the part
of Plaintiff
E. Crane operator would be liable for damages as it his duty to give warning to the plaintiff.

A

A

90
Q

The accused who was driving a jeep struck the deceased, as a result of which he sustained serious injuries. The accused put the injured person in the jeep for medical treatment, but he died. Thereafter, the accused cremated the body. The accused was charged under ss 304A(death of any person by doing any rash or negligent act not amounting to culpable homicide) and 201 of the IPC. As per s 304A, there must be a direct nexus between the death of a person and rash and negligent act of the accused that caused the death of the deceased. It was the case of the prosecution that the accused had possessed only a learner’s licence and hence was guilty of causing the death of the deceased. There was evidence to show that the accused had driven the jeep to various places on the previous day of the occurrence.
Choose Correct option:
A. Accused is guilty of Murder and Causing disappearance of evidence of offence.
B. Accused is guilty of Culpable homicide and Causing disappearance of evidence of offence.
C. Accused is guilty of Death by rash and Negligent Act and Causing disappearance of evidence of offence.
D. Accused in guilty of Culpable homicide not amount to murder and Causing disappearance of evidence of offence.
E. Accused is guilty of Causing disappearance of evidence of offence only.

A

E

91
Q

The defendants’ servant left two unattended horses in a public street. A boy threw a stone on the horses due to which they bolted and created danger for other people on the road. So, a constable came forward to protect them and suffered injuries while doing so.
Will constable get damages for the injuries suffered by him?
Choose best available option:
A. Cases of Volenti non fit injuria, no one ask him to help, So no damages
B. Rescue case, will get damages.
C. Will not get damages as it is established principle of law a consideration to do something which the promisor is already required to do is not a good consideration. For example- the public duty done by a public servant.
D. Will get damages from the servant of defendant’s
E. None of the above and will be decide by the discretion of court
i.e. Unliquidated damges.

A

B

92
Q

A bus of U.P.S.R.T.C. reached near a village where a cyclist suddenly came in front of the bus. It was heavily raining at the time and even after applying breaks the bus driver could not stop the bus as a result of this, the rear portion of the bus hit another bus. It was discovered that there was no negligence on the part of both the drivers and they tried their best in avoiding the accident. Other driver filed the suit for damages.
Choose Correct option:
A. Bus driver will be liable and pay the damages
B. Cyclist will be liable and pay the damages
C. No one will be liable
D. Principle of contributory negligence – both the drivers of bus must drive slowly with respect to to raining conditions.
E. Cyclist and the UP bus driver will be held liable for the damages

A

C

93
Q

Discounting all reservations raised by the opposition-ruled states, the Union government has fast-tracked the process to amend IAS, IPS, and IFoS service cadre rules. The move initiated early this year in January met widescale opposition from major state governments, which claimed that it will destroy the federal structure by unilaterally enacting ‘unconstitutional ‘ amendments for serving IAS officers.
According to the union minister of state for personnel Jitendra Singh, the government has raised the annual intake of IAS officers to 180 through civil services examination till CSE – 2021

  1. Rules regarding the Central deputation in the Indian Administrative Service are mentioned under _______?
    a) Rule-6 (1) of the IAS (Cadre) rules-1954.
    b) Rule - 5 (1) of the IAS (Cadre) rules 1956
    c) Rule - 9 (1) of the IAS (Cadre) rules 1954
    d) None of the above
  2. Which department is responsible for IAS services?
    a) Department of administrative reforms and public grievances
    b) Department of legal affairs
    c) Department of defense
    d) Department of Personnel and Training
  3. What is the motto of IAS Services?
    a) Excellence in action
    b) Delegated legislation
    c) Three-tier government
    d) Welfare society
  4. When were IAS services established initially?
    a) 15 August 1950
    b) 26 November 1950
    c) 26 January 1950
    d) 24 January 1950
  5. Who is known as ‘the father of civil service in India ‘?
    a) Warren Hastings
    b) Charles Cornwallis
    c) Both a and b
    d) None of the above
A

ANSWERS:

  1. A
  2. D
  3. A
    1. B
    2. B
94
Q

The Sexual Harassment of Women at Workplace (Prevention, Prohibition and Redressal) Act, 2013 is a legislative act in India that seeks to protect women from sexual harassment at their place of work. It was passed by the Lok Sabha (the lower house of the Indian Parliament) on 3 September 2012. It was passed by the Rajya Sabha (the upper house of the Indian Parliament) on 26 February 2013. The Bill got the assent of the President on 23 April 2013. The Act came into force on 9 December 2013. This statute superseded the Vishaka Guidelines for Prevention Of Sexual Harassment (POSH) introduced by the Supreme Court (SC) of India. It was reported by the International Labour Organization that very few Indian employers were compliant with this statute. Most Indian employers have not implemented the law despite the legal requirement that any workplace with more than 10 employees needs to implement it. . The government has threatened to take stern action against employers who fail to comply with this law.

  1. When was POSH (Prevention of Sexual Harassment) passed by the Lok Sabha (Lower House) as stated (1) in the passage? a) 3rd September 2012
    b) 13th September 2013
    c) 20th September 2012
    d) 26th February 2013
  2. POSH provisions are applicable to all establishments, companies, organizations (whether public or private) employing how many members? a) 14 or more
    b) 12 or more
    c) 10 or more
    d) It is not applicable yet
  3. Under which ministry was POSH launched?
    a) Ministry of Housing and Urban Affairs
    b) Ministry of Rural Development
    c) Ministry of Women & Child Development
    d) Union Minister of Skill Development and Entrepreneurship
  4. Who is the current brand ambassador of Ministry of Women and Child Development India? a) Salman Khan
    b) Aamir Khan
    c) Sachin Tendulkar
    d) Hardeep Singh Puri
  5. Which of the following statement is/are true regarding POSH (Prevention of Sexual Harassment)?
    (1) The Act defines sexual harassment at the workplace and creates a mechanism for redressal of complaints. It also provides safeguards against false or malicious charges.
    (2) Every employer is required to constitute an Internal Complaints Committee at each office or branch with 10 or more employees.

a) Only 1
b) Only 1 and 2
c) Only 2
d) Neither 1 nor 2

A
6a
7c
8d
9b
10b
95
Q

The claimant was a photographer working at a horse show. He was situated within the ring where the horse show took place (rather than behind the spectator barriers) when one of the horses galloped towards him at a significant speed after the rider lost control of it, knocking him down. The claimant sued the defendant in the tort of negligence.
1. Choose best available options:
A. Defendants are liable for not taking due care.
B. Defendants are not liable for not taking due care
C. Defendant and plaintiff both would be liable.
D. Absolute case of accident(inevitable)
E. Plaintiff would be liable for contributory negligence

A

B

96
Q

The petitioner, A, had been charged with violent robbery as part of an armed gang in 1941. He was released due to lack of evidence, but a ‘history sheet’ was opened in regard to him under the Uttar Pradesh Police Regulations. These regulations provided for surveillance powers, including powers of domiciliary visits, for habitual offenders or people likely to become criminals. Based on these provisions, the police would often visit A’s house at odd hours, waking him up when he was sleeping (to confirm that he is not on robbery or dacoity). The petitioner argued that these regulations were in violation of his right to life with dignity under Article 21 of the Indian Constitution, which includes the right to privacy. He also argued that the measures violated personal liberties guaranteed under Article 19 of the Indian Constitution.
2. Choose the correct option:
A. Petitioner argument will fail because it is statutory provision to held domiciliary visits and the purpose of statute cannot be defeated.
B. Court will held domiciliary visits unconstitutional because this impacted on the right to life, protected under Article 21 of the Constitution, which implied the right to life with human.
C. Domiciliary visits come under preventive detention, so police can restrain a crime by keeping any eye on habitual offender. So the claims of petitioner are baseless.
D. Court will order the police to not to visit in odd hours. This relief can be granted to the petitioner

A

B

97
Q

Mrs. A, a practicing advocate of the Calcutta High Court, filed a petition under Article 226 of the Constitution against the Chairman, claiming compensation for victim Smt. B, a Bangladeshi national who was gang-raped by employees of the Railways. Smt. B arrived at Calcutta and was said to wait at Ladies waiting room by a Train ticket examiner for confirmation of her berth ticket. Two men came to her, claiming to be influential persons of railway and confirmed her ticket. After that, one of those men came again and told her to accompany a boy to restaurant for food. She went for dinner and came back to ladies’ room again. When 2 another male came to her and asked her to follow her to Yatri Niwas for resting there which she doubted earlier about them but after getting confirmation by lady attendants, she accompanied them. They took her to room which was booked by the name of C where already 3 male attendants were present. B suspected something amiss when C forced her into the room. All the four men who were present inside the room brutally raped B. When she could recover, she managed to escape from the room of Yatri Niwas and came back to the platform where again she met D and found him talking to C. Seeing her condition, he pretended to help her and requested to come to his residence to rest for the night with his wife and children and assured her to help catching the following train as she missed her train while rescuing herself. Thereafter, he took B to a rented flat of his friend C and raped her. Hearing the voices from the flat landlord of the building rescued her by calling police.
3. Is this petition being maintainable under article 226/ Whether there was a relief available to the foreigner victim Smt. B by a petition filed under Article 226?
A. No, B should have approached civil court for damages and the matter should not have been considered in a petition filed under Article 226 of the Constitution, cannot be recognized.
B. No, this petition is filed as PIL. PIL is for the public interest at large and not for the individual’s interest.
C. Yes, the petition is maintainable under Article 226.
D. No, this petition is not maintainable under article 226, because you cannot approach high court or supreme court when alternative remedy is available. High Court and Supreme court will not entertain this petition when remedy is available under IPC. Moreover article 226 and 32 are for the protection of fundamental rights of citizens enshrined in Part III of the constitution.

A

C

98
Q

Harvey was interested in buying a property owned by Facey. He sent Facey a telegram stating “Will you sell us Bumper Hall Pen? Telegraph lowest cash price – answer paid.” Facey replies stating “Bumper Hall Pen £900. Harvey responded stating that he would accept £900 and asking Facey to send the title deeds. Facey then stated he did not want to sell. Harvey sued Facey alleging that telegram was an offer and he had accepted, therefore there was a binding contract.
5. Choose correct option:
A. It is a breach of contract on part of Facey
B. No valid Contract exists.
C. Harvey under mistake of fact regarding the price.
D. Contract was completed when Facey read the acceptance of Harvey to buy bumper hall pen in £900.Passage 5:
E. Consider the following statements regarding fundamental rights:

A

B

99
Q

There was an accident due to negligence of bus driver and motorcyclist. The driver and one grown up person and also a child of 4 years on the pillion got killed.
Choose the correct option regarding Compensation.
A. This is the case of contributory Negligence and the compensation of deceased persons would be deducted according to their contributory negligence.
B. Compensation for deceased persons apart from pillion driver would not be deducted by the court and the full compensation would be awarded to deceased persons.
C. Full compensation would be awarded only to child (as children are not deemed to be in contributory
D. Negligence) and for remaining two deceased it would be deducted according to their contributory negligence.
E. Full compensation would be awarded to all three as one cannot fix the contributory negligence in the case of vehicles which are apart in size and impact. (Bus vs. bike).

A

B

100
Q

Defendant company sent its deliveryman to deliver the replacement of gas cylinder to the plaintiff’s house. The cap of the cylinder was imperfect. So, the deliveryman asked the plaintiff to get something to open it. Plaintiff brings an axe for the same and handover to deliverymen, and he had opened. But the gas leaked from there and caused fire. Members from plaintiffs has died. And also, some of his neighbors also caused injuries.
Choose the best option regarding Compensation:
A. This is the case of absolute contributory Negligence and the compensation of Plaintiff would be deducted according to his contributory negligence. (Giving an axe to open cap of cylinder)
B. No Contributory negligence on part of plaintiff. Negligence was completely done by deliveryman. Full compensation will be awarded to plaintiff.
C. No Compensation would be awarded to plaintiff as it is implied knowledge that gas cylinders are risky and axe is not a tool to open the cap. Duty to take care was missing on behalf of Plaintiff. D. It was inevitable accident; no party is liable for the damage.

A

B

101
Q

The husband and wife, filed a suit against the State of Punjab, for recovery of damages to the tune of Rs.3,00,000/- on account of a female child having been born to them in spite of the wife having undergone a tubectomy operation performed by the lady surgeon. According to the plaintiffs, they already had a son and two daughters from the wed-lock lasting over 17 years. In response to a publicity campaign carried out by the Family Welfare Department of the appellant-State, wife with the consent of husband, underwent a sterilization operation on 1.8.1984. A certificate in this regard bearing mark of identification No. 505, duly signed by the lady surgeon who performed the said surgery, was issued to her. It was submitted that there was no negligence or carelessness in the performance of the surgery.
Choose the best option regarding Compensation:
A. State of Punjab is liable as per vicarious liability to pay compensation to couple on the failure of service promised.
B. Doctor who performed the operation is liable to pay compensation as failure in operation is amount to negligence in duty to take care, hence state of Punjab is not liable.
C. Neither state of Punjab nor the doctor is liable for the failure of operation. No compensation will be awarded.
D. This is the case of grave medical negligence. State of Punjab is bound to give compensation to couple and the doctor will be held liable independently.

A

C

102
Q

The complainant and 5 children of his family took an elephant Joyride, at about 8 p.m. in Mysore Dasara Exhibition Ground after having purchased the tickets for the same. After taking after taking a number of rounds while the complainants and other persons where in the process of getting down the cradle, the elephant became panicky in that Rush Hour and ran forward. The complainant was thrown on the ground as a result of which he received serious injuries resulting in total loss of eyesight of both the eyes . He was a medical practitioner. He claimed compensation of ₹.9,90,000. It was found that it was a female elephant having participated in such rides and festivals for 13 years. It had acted in film shootings, various religious functions and honoring the VIPs.
Choose Correct option:
A. Joy ride organizer will be held liable for the loss and bound to pay compensation.
B. As per Volenti non fit injuria, no compensation would be awarded.
C. As per facts there is no scope for the compensation to victim/plaintiff.
D. Compensation would be awarded after deducting amount for contributory negligence.

A

C

103
Q

The defendant, Mr. Wrench, offered to sell the farm he owned to the complainant, Mr. Hyde. He offered to sell the property for £1,200, but this was declined by Mr. Hyde. The defendant decided to write to the complainant with another offer; this time to sell the farm to him for £1,000. He made it clear that this would be his final offer regarding the property. In response, Mr. Hyde offered £950 for the farm in his letter. This was refused by Mr. Wrench and he confirmed this with the complainant. Mr. Hyde then agreed to buy the farm for
£1,000, which was the sum that h ad previously been offered.
However, Mr. Wrench refused to sell his farm. The complainant brought an action for specific performance, claiming that as Mr.
Wrench refused to sell the farm, this was a breach of contract.
Choose Correct option:
A. This is breach of contract.
B. No breach of Contract.
C. Contract can be enforced by the contract
D. No adequate consideration made by the Hyde

A

B

104
Q
Read the passage carefully and answer the following questions- 
Article 16 of the Indian constitution says-Equality of opportunity in matters of public employment 
(1)	There shall be equality of opportunity for all citizens in matters relating to employment or appointment to any office under the State 
(2)	No citizen shall, on grounds only of religion, race, caste, sex, descent, place of birth, residence or any of them, be ineligible for, or discriminated against in respect or, any employment or office under the State 
(3)	Nothing in this article shall prevent Parliament from making any law prescribing, in regard to a class or classes of employment or appointment to an office under the Government of, or any local or other authority within, a 
State or Union territory, any requirement as to residence within that State or Union territory prior to such employment or appointment 
(4)	Nothing in this article shall prevent the State from making any provision for the reservation of appointments or posts in favor of any backward class of citizens which, in the opinion of the State, is not adequately represented in the services under the State 
(5)	Nothing in this article shall affect the operation of any law which provides that the incumbent of an office in connection with the affairs of any religious or denominational institution or any member of the governing body thereof shall be a person professing a particular religion or belonging to a particular denomination. 
Answer the following with the best of your knowledge- 

Article 16(4A) of Indian Constitution, provides for which of the following? a. Catch-up rule.

b. Carry forward rule.
c. Consequential seniority.
d. All of the above.

A

C

105
Q

It is well settled practice and principle that before seizure of the property, concern taxation authority will send the notice (as required/s 154) to the concerned person defaulting tax. The appellant gave a blow to an inspector of Delhi Municipal Corporation who, in discharge of his duty, had gone to collect the milk tax (as provided under section 161 of the Corporation Act) and had seized the buffalo. The appellant was convicted for assaulting the inspector.
Consider the following statements:

a. Appellant was acting under right to private defence of property.
b. Inspector was not acting in good faith as the due notice was not served before the seizure of property.
c. Inspector was acting in good faith and under the colour of his office, though the act, may not be strictly justifiable by law.
d. Conviction of the appellant was not on merit and DMC is liable to pay the compensation to accused for its arbitrary action.

A

C